You are on page 1of 61

Sri Chaitanya IIT Academy Physics

SRI CHAITANYA IIT ACADEMY, INDIA


A.P, TELANGANA, KARNATAKA, TAMILNADU, MAHARASHTRA, DELHI, RANCHI
A Right Choice for the Real Aspirant
ICON Central Office, MADHAPUR-HYD
QUICK REVISION TEST

PARAGRAPH TYPE QUESTIONS

Paragraph: A body is projected from a point ‘P’ on the ground. x,y are its position co-ordinates at time t.
Assuming that it is projected from origin, the equations governing its motion are x=30t, y  40t  10t 2 , x and
y are in meters, t is in seconds.
1 Its time of flight is
A. 8s B. 4s C. 2s D. 1s
Answer :B
Solution : Conceptual

s
2 The maximum height reached by the body is

er
A. 80m B. 40 m C. 20 m D. 10 m

pp
Answer :B
Solution : Conceptual
to
e_
3 Its horizontal range is
A. 240 m B. 120 m C. 60 m D. 30 m
je

Answer :B
iit

Solution : Conceptual
e/

B A
.m

Paragraph: 16m
//t

Two persons B and A are separated by 16m as shown in the figure. A moves with an acceleration 1ms 2
s:

starting from rest. B moves with a constant velocity 6 ms 1


tp

4 A crosses B after time _____


ht

A. 4s B. 6s C. 8s D. 10s
Answer :C
Solution : Let A move through ‘y’. B moves through x+y in that interval.
1 1
y  at 2 x  y  ut x  ut  at 2
2 2
B x A y 1
16  6t  t 2
2
32  12t  t 2
t 2  12t  32  0
(t  8)(t  4)  0
t=4 t=8 sec
Time after which ‘B’ crosses ‘A’ is 4 seconds
Time after which ‘A’ crosses ‘B’ is 8 seconds
5 Velocity of ‘A’ when ‘B’ crosses ‘A’ is _______ ms 1
A. 6 B. 8 C. 16 D. 4
Quick Revision Test COMPREHENSION Page 1
https://t.me/iitjee_toppers
Sri Chaitanya IIT Academy Physics
Answer :D
Solution : V  1 4  4m / s

6 Velocity of ‘A’ w.r.t. B when ‘A’ crosses ‘B’ is _______ ms 1


A. 2 B. 8 C. 10 D. 16
Answer :A
Solution : VB  6ms 1 VA  8  1  8ms 1
vr  v A  vB
 8  6  2 ms 1
Paragraph: Consider a system of three equal masses and 4 pulleys arranged as shown . The mass of each
block is m

s
er
pp
m A
to
e_
B m C m
je

7 The acceleration of C is
iit

2g 2g g g
A. down words B. upwards C. upwards D. upwards
e/

23 23 4 9
.m

Answer :D
Solution :
//t
s:

8 Tension in the single thread running through all the pulleys is


tp

5 25 3 7
A. mg B. mg C. mg D. mg
ht

9 23 4 9
Answer :A
Solution : Let acceleration of A is B and C are a A , aB & aC in down ward direction
Constrain relations gives.
aA  2aC  2aB  0 …………………….(1)
Also, mg –T = ma A for A ----------------(2)
2T  mg  maB  for B ------------------(3)
2T  mg  maC  for C -----------------(4)
Paragraph: In the figure shown, p1 and p2 are massless pullies. P1 is fixed and P2 can move. Masses of
9
A,B and C are kg , 4kg  and 2 kg , respectively . All contacts are smooth and the string is massless.
32
(  37 0 , tan 37 0  0.75 & g  10ms 2 )

Quick Revision Test COMPREHENSION Page 2


https://t.me/iitjee_toppers
Sri Chaitanya IIT Academy Physics

P1

B

A P2
C
2
9 Acceleration of block ‘c’ is _________ ms
A. 1 B. 2 C. 3 D. 4
Answer :C
Solution :
10 Tension in the string is ____________
A. 10 N B. 13 N C. 24 N D. 36 N
Answer :B
Solution :

s
er
11 Normal reaction between the two blocks is ___________
A. 75 N B. 2.75 N C. 3.75 N D. 4.75 N

pp
Answer :C
Solution : Let the acceleration of B downwards be a0  a
to
e_
From constraint ; acceleration of A and C are
4a
je

a A  a cot   towards left


3
iit

a
aC  up wards
e/

2
.m

Free body diagram of A, B and C are


Ncos +T
//t

2T
s:

N1
tp

B
C
ht

N sin N sin

A
2mg mg
9m
N sin   ( a cot  ) ……………………………………..(1)
64
2mg  T  N cos   2ma ………………………………….(2)
a
2T  mg  m …………………………………….(3)
2
Solving we get
a
ac   3m / s 2
2
Ans: 3m / s 2 up wards
Paragraph: We know that when a boat travels in water, its net velocity w.r.t ground is the vector sum of two
velocities. First is the velocity of boat itself in river and other is the velocity of water w.r.t. ground.
Mathematically:

Quick Revision Test COMPREHENSION Page 3


https://t.me/iitjee_toppers
Sri Chaitanya IIT Academy Physics
  
vboat  vboat ,water  vwater
Now given that velocity of water w.r.t. ground in a river is u. Width of the river is d.
y

u
d

O A x
A boat starting from rest aims perpendicular to the river with an acceleration of a = 5t, where t is time. The
boat starts from rest from point A(1, 0) of the coordinate system as shown in figure. Assume SI units

12 Obtain the total time taken to cross the river.


A. (3d / 5)1/3 B. (6d / 5)1/3 C. (6d / 5)1/2 D. (2 d / 3)1/3
Answer :B
dv y 5t 2

s
Solution : a  5t   5t  v y 

er
dt 2

pp
dy 5 2 5
  t  y  t3
dt 2 6
5  6d 
Putting y = d, we get d  t 3  t  
1/3
to
e_

2  5 
je

13 Find the equation of trajectory of the boat.


iit

1/3 1/3 1/3 1/2


 3y   6y   6y   6y 
A. x  1    B. x  u   C. x  1  u   D. x  1  u  
e/

 5   5   5   5 
.m

Answer :C
Solution : Let at any time the position coordinates of boat be (x, y)
//t
s:
tp

(x, y)
ht

(1, 0)

5
y  t 3 , x  1  ut
6
1/3
 6y 
 x 1  u  
 5 
14 Find the drift of the boat when it is in the middle of the river
1/3 1/3 1/3 1/2
 3d   3d   6d   3d 
A. u   B. u    1 C. u   D. u  
 5   5   5   5 
Answer :A
Solution : y = d/2, drift x  1  u (3d / 5)1/3
Paragraph: A large heavy box is sliding without friction down a smooth, fixed plane of inclination  . From a
point P on the bottom of the box, a particle is projected inside the box. The initial speed of the particle with
respect to the box is u and the direction of projection makes an angle  with the bottom of the box as shown
Quick Revision Test COMPREHENSION Page 4
https://t.me/iitjee_toppers
Sri Chaitanya IIT Academy Physics
in fig.

u
 Q
P

15 Magnitude of acceleration of the particle relative to box is ____


A. g cos  B. g sin  C. g D. g cos(   )
Answer :A
box
particle
g sin 

g sin  g cos 
g
 
Solution :

s
clearly arelative = gcos only (along normal to the inclined plane)

er
16 Find the distance along the bottom of the box between the point of projection P and the point Q where

pp
the particle lands in the box. (Assume that the particle does not hit any other surface of the box. Neglect

to
air resistance).
u sin 2 u 2 sin 2 u 2 cos 2 u 2 cos 
e_
A. B. C. D.
g cos  g cos  g sin  g sin 
je

Answer: B
iit

Solution: Consider the motion of the particle along the x and y - axis as shown in fig.
e/

With respect to box, we have


.m

x
//t


s:

y P

tp

g sin 
g cos 
g sin  g
ht


  
ux  [u particle ]x  [ubox ]x  u cos   0  u cos 
  
and ax  [a particle ]x  [abox ]x
or a x  g sin   g sin   0
and u y  u sin 
a y   g cos 
Particle will hit the box after time t , then we have
1
y  u yt  a yt 2
2
1 2u sin 
or 0  (u sin  )t  ( g cos  )t 2  t 
2 g cos 
Thus distance traveled in time t inside the box
2u sin 
PQ  u cos   t  u cos  
g cos 

Quick Revision Test COMPREHENSION Page 5


https://t.me/iitjee_toppers
Sri Chaitanya IIT Academy Physics
2 2
u 2sin  cos  u sin 2
 
g cos  g cos 

17 If the horizontal displacement of the particle as seen by an observer on the ground is zero, the speed of
the box with respect to the ground at the instant when the particle was projected is
u cos(   ) u 2 cos(   ) u cos(   ) u 2 cos 
A. B. C. D.
sin  cos  cos  sin 
Answer :C
Solution : Horizontal component of initial velocity of particle w.r.t. box is u cos(   ) . Let vx is the
initial velocity of box which is down the inclined plane and v is the velocity component of box along
horizontal direction. The horizontal displacement as seen by the observer on the ground to be zero, we
have
v  u cos(   )  0 or v  u cos(   )
Then vx cos   v

s
er
v 
vx

pp

 vx 
v

u cos(   )
to
e_
cos  cos 
Paragraph: The figure shows a rod which starts rotating in horizontal plane with angular acceleration 
je

about vertical axis passing through one of its end (A). A bead of mass m just fit’s the rod and is situated at a
iit

distance ‘r’ from end A. Friction exists between rod and the bead with coefficient  . As the angular velocity
e/

of rod increases, the bead starts sliding over the rod (say after time t0 ).
.m
//t

A
s:
tp

Based on above information, answer the following questions:


ht

18 The normal force acting on bead at time t ( t0 ) is :


A. Mg B. m r ( t ) 2 C. m g 2  r 2 ( t ) 4 D. m g 2  ( r  ) 2
Answer :D
Solution : N1  mg
N 2  mr
N1
N2


90
(3dfigure)

N  N12  N 22
19 Friction force acting on bead at time t ( t0 ) is given by :

Quick Revision Test COMPREHENSION Page 6


https://t.me/iitjee_toppers
Sri Chaitanya IIT Academy Physics
A.  mg B. m r ( t )2 C.  m g 2  r 2 ( t )4 D. m g 2  (r  )2
Answer :B
Solution : Only frictional force gives the required centripetal force as it is the only force
acting along the surface or rod
 f  mr 2  mr ( t ) 2
20 If the bead start sliding at t  t0 then value of t0 is given by :

 g 2  (r  ) 2 mg
A. B.
r 2 r2

D. None of these
 g 2  (r  )2 mg
C. 2

r r 2
Answer :A
Solution : When f max  mr 2
That at this moment sliding just occurs

s
mr ( t0 ) 2   ( mg ) 2  ( mr ) 2

er
Paragraph: A tool moving in x-y plane under a quality control process under the action of various forces.

pp

One force is F    x y 2 ˆj , a force in the negative y-direction whose magnitude depends on the position of the

to
tool. The constant  is 2.50 N/m3. Consider the displacement of the tool from the origin to the point x =
e_
3.00 m, y = 3.00 m.

21 Calculate the work done on the tool by the force F if this displacement is along the straight line y = x
je

that connects these two points


iit

A. – 50.6 joule B. – 67.5 joule C. – 77.5 joule D. – 60.5 joule


e/

Answer :A
.m


Solution : w  F ( dxiˆ  dyjˆ )

//t

3
s:

w    y 3 dy ( x = y)
tp

0
3
 y4  2.5
ht

= 2.5     .(81)
 4 0 4
w  50.6 J
22 Predict the nature of the force given in the paragraph.
A. Conservative B. Non-conservative C. Can’t be predicted D. Restoring force
Answer :B
y
P
 (3,3)

A x
(3,0)
Solution :
w0 A  0
3
(33 )
wA P    .3. y 2 . dy  3  2.5
0
3

Quick Revision Test COMPREHENSION Page 7


https://t.me/iitjee_toppers
Sri Chaitanya IIT Academy Physics
= - 67.5
wOP  wO  A P
Hence the given force is non-conservative
Paragraph: A block of mass 1 kg is moving towards a movable wedge of mass 2 kg as shown in the figure.
All surfaces are smooth. When the block leaves the wedge from top, its velocity is making an angle   30
with horizontal.

v0

1.45 m
2 kg
1 kg
600

23 The value of v0 in m/s is


A. 4 B. 7 C. 10 D. 9
Answer :B
Solution : Let velocity of wedge is v1 (with respect to ground) and velocities of block is v2 (with respect

s
er
to wedge when block reaches the top of the incline.
From momentum conservation in horizontal direction.

pp
 1 v0  1 ( v2 cos 600  v1 )  2v1
v
 v0  2  3v1 ……………………(1) to
e_
2
From given condition,
je

v2 sin 60 0
tan 30 0  …………..(2)
iit

v2 cos 600  v1
e/

From energy conservation,


.m

2 2
1  v2 cos 600  v1    v2 sin 600  
  2  v12 1 v02
   1 g 1.45 …(3)
//t

2 2 2
s:

Solving above questions, we get


v0  7 m / s
tp

v1  v2  2 m / s
ht

Maximum height attained by block is,


(v sin 600 )2
H  1.45  2  1.6m
2g
Y
r
v

X
1.45 m

v1
0
60

24 To what maximum height above the base of wedge will the block rise?
A. 1.9 m B. 2.7 m C. 1.6 m D. 1.45 m
Answer :C
Solution : Let velocity of wedge is v1 (with respect to ground) and velocities of block is v2 (with respect
Quick Revision Test COMPREHENSION Page 8
https://t.me/iitjee_toppers
Sri Chaitanya IIT Academy Physics
to wedge when block reaches the top of the incline.
From momentum conservation in horizontal direction.
 1 v0  1  v2 cos 600  v1   2v1
v2
 v0   3v1 ……………………(1)
2
From given condition,
v2 sin 60 0
tan 30 0  …………..(2)
v2 cos 600  v1
From energy conservation,
2 2
1  v2 cos 600  v1  v2 sin 600 
    2 2
   2  v1  1 v0  1 g 1.45 …(3)
2 2 2
Solving above questions, we get
v0  7 m / s
v1  v2  2 m / s
Maximum height attained by block is,

s
0 2
 v sin 60 

er
2
H  1.45   1.6m
2g

pp
Y

to
r
v

e_
X
1.45 m
je
iit

v1
e/

0
60
.m

25 Let J be the magnitude of net impulse given to the block by the wedge. JH be its horizontal component
//t

and JV be its vertical component.


s:

8 B. J H  4 N .s 4 D. All of these
A. J  N .s C. JV  N .s
tp

3 3
ht

Answer :D
Solution : Let velocity of wedge is v1 (with respect to ground) and velocities of block is v2 (with respect
to wedge when block reaches the top of the incline.
From momentum conservation in horizontal direction.
 1 v0  1 ( v2 cos 600  v1 )  2v1
v
 v0  2  3v1 ……………………(1)
2
From given condition,
v2 sin 60 0
tan 30 0  …………..(2)
v2 cos 600  v1
From energy conservation,
2 2
1  v2 cos 600  v1    v2 sin 600  
  2  v12 1 v02
   1 g 1.45 …(3)
2 2 2
Solving above questions, we get
v0  7 m / s
Quick Revision Test COMPREHENSION Page 9
https://t.me/iitjee_toppers
Sri Chaitanya IIT Academy Physics
v1  v2  2 m / s
Maximum height attained by block is,
0 2

H  1.45 
 v sin 60 
2
 1.6m
2g
Y

r
v
X

1.45 m
v1
0
60

Paragraph: Two identical balls A and B, each of mass 2 kg and radius R, are suspended vertically from an
inextensible strings as shown in fig. Third ball C of mass 1 kg and radius r  ( 2  1) R falls and hits A and B
symmetrically with 10 m/s. Speed of both A and B just after the collision is 3 m/s.

s
er
1kg g

pp
c m
to
e_
R
je

2kg 2kg
iit

A B
e/

26 Speed of C just after collision is


.m

A. 2 m/s B. 2 2m / s C. 5 m / s D. ( 2  1) m / s
//t

Answer :A
Solution : As the balls A and B are constrained to move horizontally (immediately after collision). If ‘I’
s:

be the impulse imparted by ball ‘C’ to each of A and B, the impulse received by ball C from them would
tp

be 2I cos
ht


C

A
  B

Now, each of ball B and C received impulse ‘I’ as shown in Fig. 2.413, but moves horizontally as its
vertical comp. gets balanced by impulse imparted to ball B and C by the respective strings and hence,
I cos   M AVA    M BVB
For C
Initial momentum + momentum imparted = final momentum mcuc  2 J cos   mc vc -----------(1)
For A
J sin   mA vA
Quick Revision Test COMPREHENSION Page 10
https://t.me/iitjee_toppers
Sri Chaitanya IIT Academy Physics
J 6 2
From diagram   45
Substitute J equation (i) Vc = -2 m/sec.
Coefficient of restitution:
V V
e A C
uc  u A
uA  0
uC  10cos 45
3
vA 
2
VC  2
1
e
2
27 Impulse provided by each string during collision is
A. 6 2Ns B. 12 Ns C. 3 2Ns D. 6 Ns

s
Answer :D

er
Solution : As the balls A and B are constrained to move horizontally (immediately after collision). If ‘I’

pp
be the impulse imparted by ball ‘C’ to each of A and B, the impulse received by ball C from them would
be 2I cos
to
e_
je


iit

C
e/
.m

A
  B
//t
s:
tp

Now, each of ball B and C received impulse ‘I’ as shown in Fig. 2.413, but moves horizontally as its
ht

vertical comp. gets balanced by impulse imparted to ball B and C by the respec tive strings and hence,
I cos   M AVA    M BVB
For C
Initial momentum + momentum imparted = final momentum mcuc  2 J cos   mc vc -----------(1)
For A
J sin   mA vA
J 6 2
From diagram   45
Substitute J equation (i) Vc = -2 m/sec.
Coefficient of restitution:
V V
e A C
uc  u A
uA  0

Quick Revision Test COMPREHENSION Page 11


https://t.me/iitjee_toppers
Sri Chaitanya IIT Academy Physics
uC  10cos 45
3
vA 
2
VC  2
1
e
2
28 The value of coefficient of restitution is
1 1 C. 2  1 1
A. B. D.
4 2 2
Answer :D
Solution : As the balls A and B are constrained to move horizontally (immediately after collision). If ‘I’
be the impulse imparted by ball ‘C’ to each of A and B, the impulse received by ball C from them would
be 2I cos

s
er

C

pp

A
 to B
e_
je
iit

Now, each of ball B and C received impulse ‘I’ as shown in Fig. 2.413, but moves horizontally as its
e/

vertical comp. gets balanced by impulse imparted to ball B and C by the respective strings and hence,
.m

I cos   M AVA    M BVB


//t

For C
Initial momentum + momentum imparted = final momentum mcuc  2 J cos   mc vc -----------(1)
s:

For A
tp

J sin   mA vA
ht

J 6 2
From diagram   45
Substitute J equation (i) Vc = -2 m/sec.
Coefficient of restitution:
V V
e A C
uc  u A
uA  0
uC  10cos 45
3
vA 
2
VC  2
1
e
2

Quick Revision Test COMPREHENSION Page 12


https://t.me/iitjee_toppers
Sri Chaitanya IIT Academy Physics
Mg
Paragraph: In the figure shown, frictional force between the bead and the light string is . The system is
4
released from rest, with a bead of mass M at a distance  from the free end of the string as shown. Assume
string and pulley as mass less.

M bead
 

29 Acceleration of the block is


g g g g
A. downward B. upward C. upward D. downward

s
2 2 4 4

er
Answer :A

pp
to
e_
je

M bead
 
iit
e/

t
.m
//t

M
Solution :
s:

Tensions is string = friction between bob and string


tp

 mg-2t=Ma
 mg 
ht

Mg-2   Ma
 4 
g
 ablock  down
2
mg
mg 
abeed , ground  4  3 g down
m 4
a beed , string  abeed  astring
3g
 +g
4
7g
 down
4
1
sb , s  ub ,s  ( ab , s )t 2
2

Quick Revision Test COMPREHENSION Page 13


https://t.me/iitjee_toppers
Sri Chaitanya IIT Academy Physics
8l
t 
7g
30 Acceleration of the bead with respect to ground is
7g 3g g
A. downward B. downward C. g downward D. downward
4 4 2

Answer :B

M bead
 

s
er
M
Solution :

pp
Tension is string = friction between beed and string
 mg-2t=Ma
mg to
e_
Mg-2 [ ]  Ma
4
je

g
 ablock  down
iit

2
e/

mg
mg 
4  3 g down
.m

abeed , ground 
m 4
//t

a beed , string  abeed  astring


s:

3g
 +g
4
tp

7g
ht

 down
4
1
sb , s  ub ,s  ( ab , s )t 2
2
8l
t 
7g
31 Time taken by the bead to loose contact with the string is
4 8 5 3
A. B. C. D.
7g 7g 7g 7g

Answer :B

Quick Revision Test COMPREHENSION Page 14


https://t.me/iitjee_toppers
Sri Chaitanya IIT Academy Physics

M bead
 

M
Solution :
Tension in string = friction between box and string
 mg-2t=Ma
 mg 
Mg-2   Ma
 4 
g
 ablock  down

s
2

er
mg
mg 
4  3 g down

pp
abeed , ground 
m 4
a beed , string  abeed  astring
to
e_
3g
 +g
4
je

7g
iit

 down
4
e/

1
sb , s  ub ,s  ( ab , s )t 2
.m

2
8l
//t

t 
7g
s:

Paragraph: For a particle to move in a circular path, it must be provided centripetal acceleration with some
tp

force. The force could be gravitational or electromagnetic in nature.


ht

32 A vehicle of mass m moves on a horizontal circular track of friction coefficient  . Radius of the circular
track is R and the vehicle moves with constant speed V. Frictional force acting on the vehicle is:
A.  mg mV 2 mV 2 V4
B. C.  mg  D. m  2 g 2  2
R R R
Answer: B
Solution: A speed is constant, there is no tangential acceleration. Horizontal external force on the
vehicle is friction only. This provides necessary centripetal force.
mv 2
 Friction 
R
33 Let coefficient of static and kinetic frictions between the vehicle and the track mentioned in Q.No 32 be
S  0.4 and  K  0.3 respectively and the vehicle be moving with maximum speed without skidding .
The driver suddenly applies breaks hard enough to skid the wheels of the vehicle. Find the magnitude of
change in frictional force between the wheels and the road just before breaks are applied and just after
the wheels start skidding.
Quick Revision Test COMPREHENSION Page 15
https://t.me/iitjee_toppers
Sri Chaitanya IIT Academy Physics
A. 0.5 mg B. 0.1 mg C. 0.3 mg D. data insufficient
Answer: B
Solution: Just before breaks are applied, the vehicle is moving with vmax without skidding. Hence the
friction is limiting.
Flim  s mg
Just after breaks are applied, as the vehicle skids, sliding friction acts on it.
Fk   k mg
Change in friction F  Flim  Fk  (  s   k ) mg  0.1mg
Paragraph: A particle A’ strike a fixed inclined plane (making an angle  with horizontal) at point A
vertically with velocity V0 . Another particle B’ strikes the same inclined plane at point B horizontally. The
collisions are elastic. If the times of flight for the projectile like paths above the inclined plane after collisions
are same for both the particles, attempt the following questions.
A'

A
B B'

s
er
α

Find the velocity of the particle B’ :

pp
34
A. V0 tan  B. V0 cot  C. V0 sin  D. V0 cos 

to
e_
Answer :B
je

V0Cos   V ' y
iit

V0 V0 y
V sin 
e/

V 90 0 x 0
V0Cos
 90
.m

 V 
V0 sin V cos

//t

x
For Particle A ' For particle B '
s:

Solution :
tp

ax = g sin  and ay = -g cos 


1 0 V '
ht

e = 1 = u yT  a y
2 V0 cos   0
1
or V '  V0 cos  S y  u yT  a yT 2
2
1
0  (V0 cos  )T  ( g cos  )T 2
2
2V0 cos  2V0
T= 
g cos  g
Particle B’ strikes the plane horizontally a x   g sin  , a y   g cos 
Velocity of particle B’ is V
1 1
Sy = uy T + a yT 2 0 = V sin  T  g cos  T 2
2 2
2V sin  2V
T=  tan  From (1) and (2)
g cos  g
2V0 cos   2V sin  = V = V0 cot  .

Quick Revision Test COMPREHENSION Page 16


https://t.me/iitjee_toppers
Sri Chaitanya IIT Academy Physics
35 Find the range of the particle A’ :
4V02 sin  2V02 sin  2V02 cos  V02 cos 
A. B. C. D.
g g g g
Answer :A
1
Solution : R1  u xT  a xT 2
2
2
2V 1  2V  4V 2 sin 
= V0 sin   0  g sin   0  = 0
g 2  g  g
36 Find the ratio of ranges of the particles A’ and B’ :
2sin 2  2cos 2  sin 2  cos 2 
A. B. C. D.
cos 2 sin 2 cos 2 sin 2

Answer :A
1
Solution : R2  u xT  axT 2
2

s
er
2
 2V sin   1  2V sin  
= V cos     g sin   

pp
 g cos   2  g cos  
2V 2 sin  2V02 sin 
=
g

g to
e_
2V0 cos  2V02 sin 
2 2
= 
je

g sin  g
iit

2
2V0  cos 2 
= sin  
e/

2 
g  sin  
.m

 2sin 2  
R1 / R2 =  .
//t

 cos 2 
s:
tp

Paragraph: A system of two blocks is at rest. A variable horizontal force is applied on the upper block. The
ht

coefficient of friction for both contacts is  . Then

m F

2m

37 When acceleration of the upper block is 2  g , net force on the lower block by the ground is

Quick Revision Test COMPREHENSION Page 17


https://t.me/iitjee_toppers
Sri Chaitanya IIT Academy Physics
A. 3mg B.  mg C. 2  mg D. mg 9   2

Answer: D
Solution : Limiting friction on upper block (fL)1 =  mg

Limiting friction on lower block(fL)2 = 3 mg

 mg
1
N  3mg
2m
 mg

Net force = mg 9   2

38 When the acceleration of the upper block is 2 g , acceleration of lower block is

s
er
A. Zero B.  g C. 2  g g
D.

pp
2

Answer :A
to
e_
Solution : Lower block does not move for any acceleration of upper block
je

39 The net horizontal force on lower block is


iit
e/

A. Always zero B. Always  mg C.  mg towards left D. 2  mg toward left


.m

Answer :A
//t

Solution : Net force in horizontal direction on lower block is always zero as it does not move
s:
tp

Paragraph: A uniform rod of length ‘l’ and mass 2m rests on a smooth horizontal table. A point mass ‘m’
ht

moving horizontally at right angle to the rod with velocity ‘v’ collides with one end of the rod and sticks to it,
then :

40 v
The angular velocity of the system after collision is
l

v v 2v 3v
A. B. C. D.
2l l l l

Answer :B

Quick Revision Test COMPREHENSION Page 18


https://t.me/iitjee_toppers
Sri Chaitanya IIT Academy Physics
2m C

C1 V
m
Solution :

Distance of common centre of mass from ‘c’ is

mk l / 2 l
x 
3m 6

Conserve the angular momentum of system about ‘c’ before and after collision

2
l   2m  l  2m  l 2 ml 2  1
mV     
2  12 36 9 

l  ml 2 ml 2 ml 2  1

s
mV    

er
2  6 18 9 

pp
l 3 1 2  1
mV
3
 ml 2 
 18 

to
e_
l ml 2 1 V
je

mV   
3 3 l
iit
e/

1
mV 2
.m

K .Ei 
2
//t

1 V 2 1 ml 2 V 2
s:

K .E f  (3m) 
2 9 2 3 l2
tp
ht

11 11
K .E f  mV 2  mV 2
32 32

1 2
K .E f  mV 2  
2 3

41 The loss in kinetic energy of the system as a whole as a result of the collision is

mv 2 7mv 2 mv 2 2mv 2
A. B. C. D.
6 24 3 3

Answer :A

Quick Revision Test COMPREHENSION Page 19


https://t.me/iitjee_toppers
Sri Chaitanya IIT Academy Physics

Solution :

Distance of common centre of mass from ‘c’ is

mk l / 2 l
x 
3m 6

Conserve the angular momentum of system about ‘c’ before and after collision

l  (2m)l 2 (2m)l 2 ml 2  1
mV    
2  12 36 9 

l  ml 2 ml 2 ml 2  1

s
mV    

er
2  6 18 9 

pp
l  3 1  2  1
 ml 2 
mV
3  18 

to
e_
l ml 2 1 V
je

mV   
3 3 l
iit
e/

1
K .Ei  mV 2
.m

2
//t

1 V 2 1 ml 2 V 2
s:

K .E f  (3m) 
2 9 2 3 l2
tp
ht

11 11
K .E f  mV 2  mV 2
32 32

1 2
K .E f  mV 2  
2 3

Paragraph: A uniform rod of mass m and length L is free to rotate about hinge O and is vertical initially. A
slight disturbance causes the rod to rotate freely about O and it strikes the ground.

Quick Revision Test COMPREHENSION Page 20


https://t.me/iitjee_toppers
Sri Chaitanya IIT Academy Physics
42 Angular velocity with the non-hinged end of rod strikes the surface is:

A. 2gL B. 3gL C. 6gL D. gL

Answer :B
Solution : By conservation of energy

L 1 mL2 2
mg    L  3gL
2 2 3

43 Horizontal force applied on rod by hinge just before the rod hits the surface is:

A. mg mg 3mg 3mg
B. C. D.
4 4 2

Answer :D
L mL 3 gL 3mg

s
Solution : Fx  m  2   2 

er
2 2 L 2

pp
44 Vertical force applied by hinge on rod at the moment it strikes the ground is:
to
e_
A. mg mg 3mg 3mg
B. C. D.
4 4 2
je
iit

Answer :B
e/

Solution : Taking torque about O.


.m

L mL2 3g
mg    
//t

2 3 2L
s:
tp

 L  3mg 3mg mg
mg  f y  m      f y  mg  
2  4 4 4
ht

Paragraph: The vessel in the figure is closed and contains water as shown. The space between water and top
of tube is vacuum. The base area is A2  100 cm 2 and the tube area A1  10 cm 2 (take  w  1000kg / m3 ,
g  10m / s 2 )

A1  10cm2 5m


 1m
2
A2  100cm

Quick Revision Test COMPREHENSION Page 21


https://t.me/iitjee_toppers
Sri Chaitanya IIT Academy Physics
For this arrangement, answer the following questions.

45 Find the total force acting on the bottom of the vessel due to liquid.

A. 100N B. 150N C. 600N D. 300N

Answer :C
Solution : Force at the bottom  (5  1) 103 10 1N 104

46 What is the weight of the water?

A. 100N B. 150N C. 300N D. 600N

Answer :B
Solution : Wt of water = Vdg

s
Paragraph: A dumbbell is floating on water. It is observed that by attaching a point mass m (neglect its

er
volume) to the rod, at 'l' distance from the center of sphere B, the dumbbell floats with the rod horizontal on

pp
the surface of the water and each sphere exactly half submerged, as shown. Take density of water as  and

to
volume of each sphere as V. Masses of the spheres are shown in the diagram.(Neglect the mass of rod)
e_
d
je

l
iit
e/

V,M A m 2M ,V
B
.m

water surface

//t
s:
tp
ht

47 The value of m is

A. V – 3M B. 2 V – 3M C. 2 V – M D. V – M

Answer :A
Solution : V  g  (3M  m ) g

48 The value of l is

d ( V  M ) d ( V  M ) d ( V  2M ) d  V  M 
A. B. C. D.
2( V  M ) (2M  V ) 2( V  3M )  2 V  3M 
Answer :C

Quick Revision Test COMPREHENSION Page 22


https://t.me/iitjee_toppers
Sri Chaitanya IIT Academy Physics
V g  V g 
Solution : Torque about point ‘m’ is zero    2 Mg      Mg   d   
 2   2 

49 If the density of sphere A is o , the values of o in terms of  which is not possible (allowed) is

   
A.  o  B.  o  C.  o  D.  o 
6 5 4 3

Answer :D
Solution : 3Mg  V  g

 3V  0 g  V  g

 0   / 3

s
Paragraph: An infinitely long wire lying along z-axis carries a current I , flowing towards positive z-

er
direction. There is no other current. Consider a circle in x-y plane with centre at (2 m, 0, 0) and radius 1 meter.

pp
Divide the circle in small segments and let d l denote the length of a small segment in anticlockwise
direction, as shown.
to
e_
y
je
iit
e/


dl
.m

1m

I x
//t

(2,0,0)
s:
tp

50   
ht

The path integral ∮B . dl of the total magnetic field B along the perimeter of the given circle is ,

0 I 0 I C. 0 I D. 0
A. B.
8 2

Answer :D
Solution : Acc. If amp. law B = 0

51 B
 
Consider two points A (3,0,0) and B (2,1,0) on the given circle. The path integral B . dl of the total
A

magnetic field B along the perimeter of the given circle from A to B is ,
 1  1  1 D. 0
A. 0 I tan 1 B. 0 I tan 1 C. 0 I sin 1
 2 2 2 2 2
Answer :B

Quick Revision Test COMPREHENSION Page 23


https://t.me/iitjee_toppers
Sri Chaitanya IIT Academy Physics

Solution : Since ∮ B.dl  0


BDAB

BDAB

 B .dl   B.dl

BD DB

 A
I  C
D

s
er
 oI
  B.dl  (2 )   oI Tan 1 (1/ 2)   Tan 1 (1/ 2) 

pp
DB
2 

 oI 1
Tan 1   to
e_
  B.dl 
AB
2 2
je

  
iit

52 The maximum value of path integral B . dl of the total magnetic field B along the perimeter of the
e/

given circle between any two points on the circle is


.m

 I  I  I D. 0
A. 0 B. 0 C. 0
12 8 6
//t

Answer :C
s:

Solution : Since ∮ B.dl  o


tp

PQP
ht


I 0  C
Q

PQP

 oI
  B.dl   B.dl  (2 )
QP PQ
2

Quick Revision Test COMPREHENSION Page 24


https://t.me/iitjee_toppers
Sri Chaitanya IIT Academy Physics

For  Bdl to be max,  be Max, it is possible for chord OP act as tangent to the dotted circle
QP


 
6

0 I
 The maximum value if  B.dl 
PQ
6

Paragraph: To convert a galvanometer into a voltmeter, we need to calculate its internal resistance and figure
of merit. The electrical arrangement shown can be used for this purpose.

2V k1

/\/\/\/\/\
1000 

s
er
pp
G
k2
/\/\/\/\/\
to
e_
Shunt resistance
je

When k1 is closed and k 2 is open, galvanometer needle is deflected by 20 divisions which is also full scale
iit

deflection. When k 2 is also closed and 100 is taken as shunt resistance, deflection shown by the
e/

galvanometer is halved.
.m
//t
s:

53 Resistance of the galvanometer is approximately


tp

A. 100 B. 99  C. 111 D. 121


ht

Answer :C
2
Solution : I g  1
1000  G

Ig 100  2
and   100G  2
2  100  G   
   1000
 100  G 

1000
 G  111.1
9

54 Figure of merit (inverse of current sensitivity of the galvanometer) is

Quick Revision Test COMPREHENSION Page 25


https://t.me/iitjee_toppers
Sri Chaitanya IIT Academy Physics
A. 9  10 5 A / div B. 9  10 4 A / div C. 9  10 3 A / div D. 9  10 2 A / div

Answer :A
0.0018
Solution : Figure of merit = I g /    9  10 5 A / div
20

55 If the galvanometer is to be converted into a voltmeter of range 4.5 V, resistance required to be


connected to the galvanometer is

A. 2500  B. 2389  C. 2000  D. 2486 

Answer :B
Solution : V  I g (G  R )

V  GI g V

s
 R  G

er
Ig Ig

pp
R  2389

to
e_
Paragraph: A uniform solid cube of side a and mass m is suspended vertically from one of its edges as shown
figure
je
iit

O
e/

a
.m
//t
s:

56 Find the period of small oscillations


tp
ht

2 2a 2 2a 2a a/ 2
A. 2 B. 2 C. 2 D. 2
g 3g g g

Answer :B
2
Solution : Moment of inertia of the cube about O  ma 2
3

Equation of motion

2 2 a
ma .  mg. .sin  ;  angle from the vertical
3 2

3g
  
2 2a

Quick Revision Test COMPREHENSION Page 26


https://t.me/iitjee_toppers
Sri Chaitanya IIT Academy Physics

2 3g
 
T 2 2a

2 2a
Or T  2
3g

57 Find the length of equivalent simple pendulum

a B. a 2 2 2a D. 2 2a
A. C.
2 3

Answer :C
2 2a
Solution : Length of equivalent simple pendulum 
3

s
er
Paragraph: A uniform bar of mass M and length L is hanging from point S as shown in figure. The Young’s
modulus of elasticity of the bar is Y and the area of cross-section of the bar is A

pp
S to
e_
je
iit
e/

58 Find the stress at a distance x, (x<L) distance from the bottom end
.m

A. MgL / Ax B. Mgx / AL Mgx MgL


//t

C. D.
2 AL 2 Ax
s:
tp

Answer :B
ht

Solution : The weight x length of the bar is

dx
L
x

 Mg 
W  x
 L 

W Mgx
So stress at x distance from bottom 
A AL

59 Find the total elongation of bar, due to its own weight

Quick Revision Test COMPREHENSION Page 27


https://t.me/iitjee_toppers
Sri Chaitanya IIT Academy Physics
MgL 2MgL MgL 1 MgL
A. B. C. D.
AY AY 2 AY 3 AY

Answer :C
L L
Mg MgL
Solution : Total elongation in wire L  dL  L   x dx 
0
ALY 0 2 AY

Paragraph: A cylinder of radius r = 10 cm is placed between two planks as shown. Mass of the cylinder is
2kg.

10 m/s
r

4 m/s

s
60 Assuming that there is no slipping at any point, find the angular velocity of the cylinder

er
A. 3 rad .s 1 B. 30 rad .s 1 C. 300 rad .s 1 D. 0.3 rad .s 1

pp
Answer :B
to
e_
Solution : Let velocity of C.M. is vCM and angular velocity  , then for no slipping Velocity of points
of contact must be equal.
je
iit

10 m/s
e/

vCM  r
.m

r
//t

vCM  r
s:

4 m/s
tp

vCM  r  10
ht

vCM  r  4  vCM  7

3 3
 7  r  10      30 rads 1
r 10 102

61 Find K.E. of the cylinder by assuming that there is no slipping at any point

A. 44.5 J B. 49 J C. 53.5 J D. 50 J

Answer :C
1 2 1 1 1 2 r2
Solution : K .E.  mvCM   CM  2   2  7 2   2
2 2 2 2 2

Quick Revision Test COMPREHENSION Page 28


https://t.me/iitjee_toppers
Sri Chaitanya IIT Academy Physics

1 0.1  30  30  0.1
 49  (0.2) 2 (30) 2  49   49  4.5  53.5 joule
2 2

Paragraph: Two uncharged capacitors A and B, each of capacitance C, and an ideal inductor of inductance L
are connected to a battery of emf E as shown in the adjacent circuit diagram. The switches S1 and S 2 are
operated in two stages as follows:

State 1: At time t = 0, switch S1 is closed while switch S 2 remains open.

 
Stage 2: At time t  t0  LC   , switch S1 is opened and switch S 2 is closed.
2

A B

s
er
E L

pp
to
e_
S1 S2
je

62 The charge on capacitor A at time t  t0 is


iit
e/

CE B. CE CE D. 2CE
A. C.
.m

2 4
//t

Answer :B
s:

q LdI q
Solution : Enet  E 
tp

C dt C
ht

dI q d 2q 1
 E  2  (q  CE )
dt C dt LC

dq
 q  CE  ASin(t   )  q  CE  A sin(t   ) and i   A cos(t   )
dt

1
with t = 0, q = 0, i = 0,  =
LC

 q  CE (1  cos t )


At t  t0  , q0  CE
2

Quick Revision Test COMPREHENSION Page 29


https://t.me/iitjee_toppers
Sri Chaitanya IIT Academy Physics
63 The current flowing through the inductor at t  t0 is

C C 1 C D. Zero
A. E B. 2 E C. E
L L 2 L

Answer :A

Solution : i  CE Sint , t 
2

1 C
 i0  CE  CE  E
LC L

64 The maximum current through the inductor in stage 2 is

 3C   5C   C   C

s
A.  B.  C.  D. 
 2 L  E  4 L  E  5 L  2 E  L  E

er
       

pp
Answer :A

to
Solution : At time t  t0 , let the charge on B is q and that on A will be CE-q and I is the current in the
e_
circuit.
je

1 q02 1 2 1 2 1 q 2 1 (CE  q)2


iit

 Li0  Li  
2 C 2 2 2C 2 C
e/
.m

1 C 2E2 1 C 1 q 2 C 2 E 2 q 2 2CEq
   L  E 2  Li 2    
2 C 2 L 2 2C 2C 2C 2C
//t
s:

1 2 q 2 CE 2
 CE 2  Li    Eq
tp

2 C 2
ht

1 2 1 q2
Li  CE 2   Eq
2 2 C

C 2 2q 2 2 Eq
i2  E  
L CL L

di
i is max, when 0
dt

2 2E
0 (2 qi )  i0
CL L

CE
q
2

Quick Revision Test COMPREHENSION Page 30


https://t.me/iitjee_toppers
Sri Chaitanya IIT Academy Physics
2
2 CE 2 2  CE  2 E  CE 
imax       
L CL  2  L  2 

3 CE 2  3C 
imax     E
2 L  2L 

Paragraph: A tank is filled with a liquid of density  up to a height H. Water flows out from the tank
through a nozzle of uniform cross-sectional area ‘a’. The exit of the nozzle makes angle 45 with the
horizontal. At a given instant, the tank rests in equilibrium against a compressed spring of force constant k.
The top of the tank is open to the atmosphere of pressure P0 . The cross-sectional area of the tank is very large
H
in comparison to that of the nozzle. The exit of the nozzle is at a height from the base of the tank.
4

s
er
pp
H
k to 45
e_
H
A 4
je
iit
e/

65 The compression of the spring at the given instant is


.m

 agH 3 2  agH 4 agH 3 agH


//t

A. B. C. D.
3 2k 8k 3 2k 2 2k
s:
tp
ht

Answer :D
3H 1 1 3 agH  2 3H 
Solution :  av 2 cos 45  kx   a  2 g  .  x x  v  2g  
4 2 k 2 2k  4 

66 The pressure inside the nozzle just after the entry point (at A) is

A. P0  H  g H C. P0  H  g 3
B. P0  g D. P0   gH
4 4

Answer :B
1 2 H 1 H
Solution : p A   v  p0   g   v 2  pa  P0   g
2 4 2 4

67 The maximum height, from the base of the tank, reached by the liquid coming out of the nozzle at the

Quick Revision Test COMPREHENSION Page 31


https://t.me/iitjee_toppers
Sri Chaitanya IIT Academy Physics
given instant is

3H 3H 5H 3H
A. B. C. D.
4 2 8 8 4

Answer :C
v 2 sin 2 45 H 3H 1 1 H 5 H
Solution : hmax    2g     
2g 4 4 2 2g 4 8

Paragraph: A thin ring of radius R meters is placed in x-y plane such that its centre lies on origin. The half
ring in region x < 0 carries uniform linear charge density   C/m and the remaining half ring in region x > 0
carries uniform linear charge density   C / m .(Take v = 0 at infinity)

y
+++
+
+

s
+
+

er
+
x +
y
+ +λ -λ

pp
+
+
+

to
+ +
e_
y
je

68  R 
The electric potential (in volts) at point P whose coordinates are  0 m,  m  is
iit

 2 
e/

1  1 
.m

A. B. 0 C. D. cannot be determined
4πε 0 2 4πε 0 4
//t
s:

Answer :B
tp

Solution : Consider two small elements of ring having charges +dq and –dq symmetrically located about
ht

y-axis

The potential due to this pair at any point on y-axis is zero.

The sum of potential due to all such possible pairs is zero at all points on y-axis. Hence potential at
 R
P  0,  is zero
 2

y
+dq x x x - - -dq
x
x --
x θ θ -
x --
x dθ
dθ --
x' x
x +λ -λ
x
--

x --
x
x --
xx
xx

Quick Revision Test COMPREHENSION Page 32


https://t.me/iitjee_toppers
Sri Chaitanya IIT Academy Physics
69  R 
The direction of electric field at point P whose coordinates are  0 m,  m  is
 2 

A. along positive x-direction B. along negative x-direction

C. along negative y-direction D. none of these

Answer :A
Solution : Since all charge lies in x-y plane, hence direction of electric field at point P should be in x-y
plane

Also y-axis is an equipotential (zero potential) line. Hence direction of electric field at all point of y-axis
is should be normal to y-axis.

The direction of electric field at P should be in x-y plane and normal to y-axis. Hence direction of
electric field is along positive x direction

s
70 Dipole moment of the ring in C . m is

er
A.  (2πR 2  )iˆ B. (2πR 2  )iˆ C.  (4R 2  )iˆ D. (4R 2  )iˆ

pp
Answer :C
to
Solution : Consider two small elements of ring having charge +dq and –dq as shown in figure.
e_
je

y
xx x - - - -dq
iit

x --
x
e/

x
x -
x dθ --
.m

x' xx θ - x
--

+λ x
//t

x
--
x -- -λ
x
+dq x x x
s:
tp

y
ht

The pair constitutes a dipole of dipole moment.

dp  dq 2R

 (λRdθ)2R

The net dipole moment of system is vector sum of diplole

moments of all such pairs of elementary charges.

By symmetry the resultant dipole moment is along negative x-direction.

Net dipole moment

Quick Revision Test COMPREHENSION Page 33


https://t.me/iitjee_toppers
Sri Chaitanya IIT Academy Physics
+π/2

  (dp cos ) i
-π/2

+π/2
2

  (2λR cos  dθ) i
-π/2


 4R 2 i

Paragraph: Three point charges + 3Q, + 2Q and – Q respectively are located at distances each "a" from the
1
origin as shown in the fig. (k  )
4 0

+ 3Q

y-axis
a

s
er
x-axis
-Q a O a + 2Q

pp

71
to
At the origin, the magnitudes of x and y-components of the net electric field E are
e_
k3Q k3Q 1 k3Q k3Q
A. Ex  2E y  B. Ex  E y  C. Ex  E y  2 D. E x  3 E y 
a2 a2 a2
je

2 a
iit

Answer :B
e/

1 2Q ˆ
Solution : E2  . ( i )
.m

4 0 a 2
//t

2Q ˆ
s:

= K ( i )
a2
tp
ht

1 Q
E1 
4 w a ( iˆ)
2

3
3Q

E1 E2
1 P
2
Q a a
2Q
E3

Quick Revision Test COMPREHENSION Page 34


https://t.me/iitjee_toppers
Sri Chaitanya IIT Academy Physics
KQ ˆ
= (i )
a2

E  E1  E 2 E 4  E 3

1 3Q
E3 
4 0 a ( iˆ)
2

K (3Q)(Q) K (2Q)( Q)


Potential energy of system, u = 
2a 2a

KQ L 3
U [ 1 ]
a 2

72 the electric potential energy 'U' for the configuration of the three charges is

s
er
kQ 2  1  kQ 2  1  kQ 2  3  kQ 2  3 
A.  1  B.   1 C.   1 D.  1
a  a  2  a  2  a  2 

pp
2

Answer :D
to
e_
1 2Q ˆ
Solution : E2  . ( i )
4 0 a 2
je

2Q ˆ
iit

= K ( i )
a2
e/

1 Q
.m

E1 
4 w a (iˆ)
2
//t

3
s:

3Q
tp
ht

E1 E2
1 P
2
Q a a 2Q
E3

KQ ˆ
= (i )
a2
E  E1  E 2
E 4  E3
1 3Q
E3 
4 0 a (iˆ)
2

Quick Revision Test COMPREHENSION Page 35


https://t.me/iitjee_toppers
Sri Chaitanya IIT Academy Physics
K (3Q)(Q) K (2Q)( Q)
Potential energy of system, u = 
2a 2a

KQ L  3 
U  
a  2 1 

Paragraph: A thin cylindrical shell closed at both ends is subjected to a uniform excess internal pressure P
over outside. The wall thickness is h and the inner radius is r ( h  r ) . Y = Young’s modulus of material of
the cylinder.

73 The longitudinal normal stress existing in the walls due to P is


Pr Ph Pr
A. P B. C. D.
h r 2h
Answer :D

s
p. r 2 pr

er
Solution : The normal stress  
2 rh 2 h

pp
74 The circumferential normal stress existing in the walls due to P is
Pr Ph to Pr
e_
A. P B. C. D.
h r 2h
je

Answer :B
iit

p 2 RL pA
Solution : Circum normal stress = 
e/

2 Lh h
.m

75 The increase in radius of the cylinder is (Poisson’s ratio is  )


//t

Pr 2
  Ph 2 Pr 2   Pr 2
A. 1  B. 1  2  C. 1  2  D. 1  2 
s:

Yh 2  Yh Yh Yh
tp

Answer :A
ht

Solution : Circum strain due to circum stress  .


pr 2R
 
Yh 2 r
2
pr
r 
Yh
r 
Circum strain due to long. Stress 
r 
pr 2
r  
2 yh
pr 2  
(r ) net  1  2 
yh

Paragraph:

A body of mass m falls from a height h onto a pan of negligible mass and lying on a light vertical spring of

Quick Revision Test COMPREHENSION Page 36


https://t.me/iitjee_toppers
Sri Chaitanya IIT Academy Physics
force constant K. The mass sticks to the pan and executes simple harmonic motion in the vertical Line.

u=0
m

reference level (RL)


(gravitational PE = 0)
K

76 The mean position of oscillations is

A. at the original reference level of the pan mg


B. above the RL by
K

s
mg 2mg
C. below the RL by D. below the RL by

er
K K

pp
Answer :C
Solution : x0 
mg
to
e_
k
je
iit

77 The amplitude of oscillations is


e/

mg 2kh mg kh mg 2 kh mg  2hk 
.m

A. 1 B. 1 C. 1 D.  1  1
K mg K mg 2K mg K  mg 
//t

Answer :A
s:

1 2
tp

Solution : mg ( h  x )  kx  kx 2  2m gx  2mgh  0
2
ht

mg mg 2kh
x  1 =maximum compression of spring
k k mg

mg
Amplitude A  x 
k

mg 2hk
1
k mg

78 The total energy of oscillations is

m2 g 2 m2 g 2 m2 g 2
A. mgh  B. mgh C. D. mgh 
K 2K 2K

Quick Revision Test COMPREHENSION Page 37


https://t.me/iitjee_toppers
Sri Chaitanya IIT Academy Physics
Answer :D
Solution : It is the energy that changes between PE and KE during oscillation. It is equal to KEmax

1 2 k m 2 g 2  2hk 
E  kA  . 2  1  
2 2 k  mg 

m2 g 2 2hkmg
 
2k 2k

m2 g 2
 mgh 
2k

Paragraph: A pulse is started at a time t = 0 along the +x direction on a long, taut string. The shape of the
x

s
 1 for  4  x  0

er
4
pulse at t = 0 is given by function f(x) with f ( x)  { Here f and x are in
 x  1 for 0  x  1

pp
0 otherwise

to
centimeters. The linear mass density of the string is 50 g/m and it is under a tension of 5N.
e_
je

79 The shape of the string is drawn at t = 0 and the area of the pulse enclosed by the string and the x-axis is
iit

measured. It will be equal to


e/

A. 2 cm 2 B. 2.5 cm 2 C. 4 cm 2 D. 5 cm 2


.m

Answer :B
//t

Solution : Shape of the pulse is


s:
tp

1
ht

4 1

1
area made by pulse with x-axis = (5)1  2.5cm 2
2

80 The vertical displacement of the particle of the string at x = 7 cm and t = 0.01 s will be

A. 0.75 cm B. 0.5 cm C. 0.25 cm D. Zero

Answer :C
Solution : Shape of the pulse is

Quick Revision Test COMPREHENSION Page 38


https://t.me/iitjee_toppers
Sri Chaitanya IIT Academy Physics

4 1

At t = 0.015 shape of pulse is

x = (1000)(0.01) = 10cm

4 1

s
er
pp
y 1
at x = 7cm,   y  0.25cm
1 4
to
e_
81 The transverse velocity of the particle at x = 13 cm and t = 0.015 s will be
je

A. -250 cm/s B. -500 cm/s C. 500 cm/s D. -1000 cm/s


iit
e/

Answer :A
.m

Solution : Shape of the pulse is


//t

1
s:
tp
ht

4 1

At t = 0.015 sec pulse is at and x = 13cm is

v p  ( slope)vw

11 13 1516

Quick Revision Test COMPREHENSION Page 39


https://t.me/iitjee_toppers
Sri Chaitanya IIT Academy Physics
1
   1000  250cm / s
4

Paragraph: An Indian submarine is moving in “Arab Sager” with a constant velocity. To detect enemy it
sends out sonar waves which travel with velocity 1050 m/s in water. Initially the waves are getting reflected
from a fixed island and the reflected waves are coming back to submarine. The frequency of reflected waves
are detected by the submarine and found to be 10% greater than the sent waves.

Sonar waves
Insian
submarine

Fixed
Island

Now an enemy ship comes in front. Due to which the frequency of reflected waves detected by submarine
becomes 21% greater than the sent waves.

s
er
82 If the wavelength received by enemy ship is 1 and wavelength of reflected waves received by

pp
 
submarine is 2 then  1  equals
 2 
to
e_
A. 1 B. 1.1 C. 1.2 D. 2
je
iit

Answer :B
e/

Solution : v1  velocity of submarine


.m

v  velocity of sound
//t
s:

v2  velocity of enemy ship


tp

Case : submarine moves towards fixed island


ht

Case (i) submarine approaches an island and receives a frequency

 v  v1 
f1   f Solving it v1  50m / s
 v  v1 
110  1050  v1 
f  f
100  1050  v1 

Case (ii) submarine approaches an enemyship

121  v  v2   v  v1 
f 11  f   f
100  v  v2   v  v1 

Quick Revision Test COMPREHENSION Page 40


https://t.me/iitjee_toppers
Sri Chaitanya IIT Academy Physics
 v2  50

2
 1050  50 
v  
 1050  50 
1 1 11
 f f
11
  1  1050  50  1.1
 v f 1050  50
f 11

83 Bulk modulus of sea water should be approximately   water  1000 kg / m3 

A. 1.1 108 N / m 2 B. 1.1 109 N / m 2 C. 1.1 1010 N / m 2 D. 1.1 1011 N / m 2

Answer :B
Solution : v1  velocity of submarine

v  velocity of sound

s
er
v2  velocity of enemy ship

pp
Case : submarine moves towards fixed island
to
e_
Case (i) submarine approaches an island and receives a frequency
je

 v  v1 
iit

f1   f Solving it v1  50m / s
e/

 v  v1 
.m

110  1050  v1 
f  f
100  1050  v1 
//t
s:

Case (ii) submarine approaches an enemy ship


tp

121 v  v2 v  v1
ht

f 11  f ( )( )f
100 v  v2 v  v1

 v2  50

k
v

 k   v 2  1000  (1050) 2  1.1 109

Paragraph: A system of men and trollies is shown in figure. To the left end of the string, a trolley of mass M
is connected on which a man of mass m is standing. To the right end of the string, another trolley of mass m is
connected on which a man of mass M is standing. Initially the system is at rest. All of a sudden, both the men
leap upwards simultaneously with the same velocity u w.r.t. ground.

Quick Revision Test COMPREHENSION Page 41


https://t.me/iitjee_toppers
Sri Chaitanya IIT Academy Physics

. .
M
m M
m
84 Find the relative velocity of left man with respect to his trolley just after he leaps upwards.

mu Mu 2mu 2Mu
A. B. C. D.
mM mM mM mM

Answer :C

s
Solution : I1  mu, I 2  Mu ,

er
pp
I1  I  Mu0 , I  I 2  mu0

to
. .
e_
je
iit
e/

m u u M
.m

I I1 I2 I u0
//t

M m
s:

I1 I2
u0
tp
ht

( m  M )u
Solve to get : u0 
mM

Relative velocity of left man w.r.t his trolley :

(m  M )u 2mu
u  u0  u  
mM mM

85 Find the impulse generated in the string connecting the trollies during this process.

Mmu ( M 2  m 2 )u m2u M 2u
A. B. C. D.
mM mM mM mM

Answer :B

Quick Revision Test COMPREHENSION Page 42


https://t.me/iitjee_toppers
Sri Chaitanya IIT Academy Physics
M (m  M ) (m2  M 2 )u
Solution : I  I1  Mu0  mu  Mu0  mu  u
mM M m

86 What is the correct statement among the following? Assume that trollies move with deceleration after
jumping of men.

A. When the men are at the highest points of their motion, the trolleys will also be instantaneously at
rest.

B. When the men are at the highest points of their motion, the left trolley will be moving downward.

C. Impulses acting on both the men will be same in the given process.

D. None of the above are true

Answer :A
Solution : Time taken for men to come to rest :

s
er
u

pp
t
g

Acceleration of trolley (after the men have jumped): to


e_
je

(m  M )
a g
mM
iit
e/

Time taken by trolleys to come to rest :


.m

u0 (m  M )u (m  M ) u
//t

t1     t
a mM (m  M ) g g
s:
tp

Paragraph: A uniform cylinder of radius R and mass m is spines about its axis with angular velocity 0 and
ht

then placed at a corner, formed with rough horizontal floor and rough vertical wall as shown in figure. The
coefficient of friction between the contact surfaces and the cylinder is k .

0

87 The normal reaction imparted by the wall on the cylinder is

2 k mg k mg  k mg k mg
A. B. C. D.
1  k2 1  k2 1  2  k2 2   k2

Quick Revision Test COMPREHENSION Page 43


https://t.me/iitjee_toppers
Sri Chaitanya IIT Academy Physics
Answer :B

N1
N 2
0

N2

N1

mg
Solution :

As the centre of mass of the cylinder does not accelerate,

N 2  k N1  0 i 

N1  k N 2  mg  0  ii 

s
er
mg  mg

pp
On solving these equations we get N1  2
, N2  k 2
1  k 1  k

Torque on the cylinder about the axis of rotation to


e_
je

k 1   k 
 in  mg  0  N1  0  N 2  0   k N1R  k N 2 R  mgR
iit

1  k2
e/

k 1   k 
.m

1 
Torque equation about its axis is   I  2
mgR   mR 2  
1  k 2 
//t
s:

2  k 1  k  g
 
tp

1    R
2
k
ht

Using equation  2  02  2 ,

 2  1    g 
O  02   k k
 2
 1  k  R 
2

(  and  are in opposite sense)


1     R
2
k
2
0

4 k 1  k  g

Number of rotations accomplished is

Quick Revision Test COMPREHENSION Page 44


https://t.me/iitjee_toppers
Sri Chaitanya IIT Academy Physics

N


1  k2  02 R  1  k2  02 R
2 2  4  k 1  k  g 8k 1  k  g

88 Angular acceleration of the cylinder is

k 1   k  k k 1  k  g
A. clockwise direction B. anticlockwise direction
1  k2  R 1  k2  R
2 k 1  k  g 2 k 1  k  g
C. clockwise direction D. anticlockwise direction
1    R
2
k 1    R
2
k

Answer :C
N1
N 2
0

s
er
N2

pp
N1

Solution :
mg
to
e_
je

As the centre of mass of the cylinder does not accelerate,


iit

N 2  k N1  0 i 
e/
.m

N1  k N 2  mg  0  ii 
//t
s:

mg  mg
On solving these equations we get N1  , N2  k 2
1   k2
tp

1  k
ht

Torque on the cylinder about the axis of rotation

k 1   k 
 in  mg  0  N1  0  N 2  0   k N1R  k N 2 R  mgR
1  k2

k 1   k  1 
Torque equation about its axis is   I  2
mgR   mR 2  
1  k 2 

2  k 1  k  g
 
1    R
2
k

Using equation  2  02  2 ,

Quick Revision Test COMPREHENSION Page 45


https://t.me/iitjee_toppers
Sri Chaitanya IIT Academy Physics
 2  1  k  g 
O  02   k  2
 1  k  R 
2

(  and  are in opposite sense)


1     R
2
k
2
0

4 k 1  k  g

Number of rotations accomplished is

N


1  k2  
1  k2  02 R
2 2  4  k 1  k  g 8k 1  k  g

89 How many rotations will the cylinder accomplish before it stops?

s
1     R
2 2
1     R
2 2
8 1  k2  02 R 3 1  k2  02 R

er
k 0 k 0
A. B. C. D.
k 1  k  g 8k 1  k  g k 1  k  g k 1  k  g

pp
Answer :B to
e_
N1
N 2
je

0
iit

N2
e/

N1

mg
Solution :
.m
//t

As the centre of mass of the cylinder does not accelerate,


s:

N 2   k N1  0 i 
tp
ht

N1  k N 2  mg  0  ii 
mg  mg
On solving these equations we get N1  2
, N2  k 2
1  k 1  k

Torque on the cylinder about the axis of rotation

k 1   k 
 in  mg  0  N1  0  N 2  0 k N1R  k N 2 R  mgR
1  k2

k 1   k  1 
Torque equation about its axis is   I  2
mgR   mR 2  
1  k 2 

Quick Revision Test COMPREHENSION Page 46


https://t.me/iitjee_toppers
Sri Chaitanya IIT Academy Physics
2  k 1  k  g
 
1    R
2
k

Using equation  2  02  2 ,

 2  1  k  g 
O  02   k 2 
 1  k  R
2


(  and  are in opposite sense)


1     R
2
k
2
0

4  k 1  2  g

Number of rotations accomplished is

s
1  k2  02 R 1   k2  02 R

er

N  

pp
2 2  4  k 1  k  g 8k 1  k  g

to
Paragraph: Light having photon energy h v is incident on a metallic plate having work function
e_
 to eject t he electrons. The most energetic electrons are then allowed to enter in a region of
je

uniform magnetic field B as shown in Fig. The electrons are projected in X-Z plane making an

angle  with X-axis and magnetic field is B  B0iˆ along X-axis.
iit
e/
.m

Y
//t
s:
tp
ht

B
X

Z
e-1

Maximum pitch of the helix described by an electron is found to be p. Take mass of electron as
m and charge as q. Based on above information, answer the following questions:

90 The correct relation between p and B0 is

A. qpB0  2 cos  2(hv   )m 2(hv   )


B. qpB0  2 cos 
m

Quick Revision Test COMPREHENSION Page 47


https://t.me/iitjee_toppers
Sri Chaitanya IIT Academy Physics

C. pqB0  2 2(hv   )m 2 m
D. p  hv  
qB0

Answer :A
Solution : At any time t the location of electron is shown as P. In two dimensional view of electron in
YZ- plane, the situation is more clear.

2 KE 2(hv   )
v 
m m
2 m
p  v cos 
qB0
2(hv   )
pqB0  2 cos  m
m
 2 cos   2 m(hv   )

s
Y

er
Y

pp
O
Z

to
P
X
t
e_
P
je

Z
iit
e/

X-coordinate, x  v cos   t
.m

Y-coordinate, y  [ R  R cos t ]
//t
s:

Z-coordinate, z  R sin t
tp
ht

mv sin   qB 
So, z   sin  0 t 
qB0  m 

2m(hv   )  sin   qB t 
  sin  0 
qB0  m 

2( hv   )
From x  v cos   t   cos   t
m

As v increases, slope of x versus t graph (a straight line) increases.

91 Considering the instant of crossing origin at t = 0, the Z-coordinate of the location of electron as a
function of time is

Quick Revision Test COMPREHENSION Page 48


https://t.me/iitjee_toppers
Sri Chaitanya IIT Academy Physics

2m ( hv   )   qB t   2m ( hv   )  qB t 
A.  (sin  ) 1  cos  0   B. (sin  ) sin  0 
qB0   m  qB0  m 

 2m(hv   )  qB t  2m( hv   )  qB t 
C. (sin  )sin  0  D. sin  0 
qB0  m  qB0  m 

Answer :B
Solution : At any time t the location of electron is shown as P. In two dimensional view of electron in
YZ- plane, the situation is more clear.

2 KE 2(hv   )
v 
m m
2 m
p  v cos 
qB0
2(hv   )
pqB0  2 cos  m

s
m

er
 2 cos   2 m(hv   )

pp
Y
to
Y
e_
O
je

Z P
t
iit

X
e/

P
.m

Z
//t

X-coordinate, x  v cos   t
s:
tp

Y-coordinate, y  [ R  R cos t ]
ht

Z-coordinate, z  R sin t

mv sin   qB 
So, z   sin  0 t 
qB0  m 

2m(hv   )  sin   qB t 
  sin  0 
qB0  m 

2( hv   )
From x  v cos   t   cos   t
m

As v increases, slope of x versus t graph (a straight line) increases.

92 The plot between X-coordinate of the location of electron as a function of time for different frequencies

Quick Revision Test COMPREHENSION Page 49


https://t.me/iitjee_toppers
Sri Chaitanya IIT Academy Physics
v of the incident light, is

x v1  v2 x x v1  v2 x
v2  v1 v1  v2
v1 v2 v1 v2 v1 v1
v2

v2
t B. t t D. t
A. C.

Answer :C
Solution : At any time t the location of electron is shown as P. In two dimensional view of electron in
YZ- plane, the situation is more clear.

2 KE 2(hv   )
v 
m m
2 m
p  v cos 

s
qB0

er
2(hv   )

pp
pqB0  2 cos  m
m
 2 cos   2 m(hv   )
to
e_
Y
je

Y
iit

O
e/

Z P
t
.m

X
P
//t
s:

Z
tp

X-coordinate, x  v cos   t
ht

Y-coordinate, y  [ R  R cos t ]

Z-coordinate, z  R sin t

mv sin   qB 
So, z   sin  0 t 
qB0  m 

2m (hv   )  sin   qB t 
  sin  0 
qB0  m 

2(hv   )
From x  v cos   t   cos   t
m

As v increases, slope of x versus t graph (a straight line) increases.


Quick Revision Test COMPREHENSION Page 50
https://t.me/iitjee_toppers
Sri Chaitanya IIT Academy Physics
Paragraph: A standing wave y = 2A sinkx cos t is setup in the wire AB fixed at both ends by two vertical
walls (see the figure). The region between the wall contains a constant magnetic field B. Now answer the
following questions.

x
x x x x x
x x x x x x
A x x x x x B
x x x x

93 The wire is found to vibrate in the 3rd harmonic. The maximum emf induced is

4AB 3AB 2AB AB


A. B. C. D.
k k k k

Answer :A

s
Solution : Induced emf is maximum when the elements are crossing the mean position.

er
pp
  
2 2 2
emax  B (dx)2 A sin kx  B dx 2 A sin kx  B dx.2 A sin kx
0 0 0
to
e_
4 BA

je

k
iit

94 In the above question, the time when the emf becomes maximum for the first time is
e/
.m

2   
A. B. C. D.
  2 4
//t
s:

Answer :A
Solution : v  2 A sin kx sin t emf is max when v is maximum
tp
ht


sin t  1  sin
2
 
t   t 
2 2

95 In which of the following modes the emf induced in AB is always zero?

A. Fundamental mode B. Second harmonic C. Second overtone D. Fourth overtone

Answer :A
Solution : Second harmonic

Paragraph: Experiments show that the number of nucleous (protons and neutrons) per unit volume inside a
nucleus is fairly constant near its center, and gradually decrease in the outer region. The near constant of
nucleon density results from the fact that each nucleon in a nucleus interacts only with a small number of

Quick Revision Test COMPREHENSION Page 51


https://t.me/iitjee_toppers
Sri Chaitanya IIT Academy Physics
nucleons in its surrounding through attractive nuclear force. The nucleon density  ( r ) as a function of the
0
distance r from the center of the nucleus is approximately given by  (r )  Where 0  0.17
r  R
1  exp 
 a 
nucleon/ fm3 , R  (1.1 fm) A1/3 and a  0.55 fm Here A = mass number.

96 The mass density in a nucleus near its center, in units of kg/ m3 , is in the range

A. 1015 to1020 B. 105 to1010 C. 1010 to1015 D. 1020 to1025

Answer :A
Solution : Density at the centre

0 0
 
R 1  e 2

s
a

er
pp
1+e


0.17
nucleons/ fm3 to
e_
1.135
je

0.15 1027
 1.5 1017 kgm 3
iit


10 45
e/
.m

216
97 For the nucleus Te, the value of r for which the nucleon density falls to half its value at the centre is
in the range
//t
s:

A. 7 to 8 fm B. 5 to 6 fm C. 6 to 7 fm D. 8 to 9 fm
tp

Answer :C
ht

0 0
Solution : 
r 
 12 
a 
2(1  e12 )
1 e

r 
 12 
1  e a 
 2(1  e12 )  e 12 [e r / a  2]  1

r
( er / a  2)  e12   12
a

(neglecting of no comparison to e12 )

 r  12 a  0.55  12 fm  6.6 fm

28
98 The plats of  ( r ) versus r for Si and another nucleus X are shown in the figure. The nucleus X could

Quick Revision Test COMPREHENSION Page 52


https://t.me/iitjee_toppers
Sri Chaitanya IIT Academy Physics
be
42 94 63 142
A. Ca B. Zr C. Cu D. Ba

Answer :D
0 0
Solution : ( r / a 10)

1 e 2(1  e 10 )

1  e( r / a 10)  2(1  e 10 )

e 10 (er / a  2)  1  (er / a  2)  e10  r  10a  5.5 fm

142
Nearest value of A is Ba

Paragraph: A dense collection of equal number of electrons and positive ions is called neutral plasma.
Certain solids containing fixed positive ions surrounded by free electrons can be treated as neutral plasma.

s
Let ‘N’ be the number density of free electrons, each of mass ‘m’, When the electrons are subjected to an

er
electric field, they are displaced relatively away from the heavy positive ions. If the electric field becomes

pp
zero, the electrons begin to oscillate about the positive ions with a natural angular frequency '  p ' which is
called the plasma frequency. To sustain the oscillations, a time varying electric field needs to be applied that
to
has an angular frequency  , where a part of the energy is absorbed and part of it is reflected. As 
e_
approaches P , all the free electrons are set to resonance together and all the energy is reflected. This is the
je

explanation of high reflectivity of metals.


iit
e/
.m
//t
s:

99 Taking the electronic charge as ‘e’ and the permittivity as '  0 ' , use dimensional analysis to determine
tp

the correct expression for  p .


ht

Ne m 0 Ne2 m 0
A. B. C. D.
m 0 Ne m 0 Ne 2
Answer :C
Solution :

100 Estimate the wavelength at which plasma reflection will occur for a metal having the density of
electrons N  4  10 27 m 3 . Take  0  10 11 and m  10 30 , where these quantities are in proper SI units.
A. 800 nm B. 600 nm C. 300 nm D. 200 nm
Answer :B
Solution :

Paragraph: When subatomic particles undergo reactions, energy is conserved, but mass is not necessarily
conserved. However, a particle’s mass ‘contributes’ to its total energy, in accordance with Einstein’s famous
equation, E  mc 2 . In this equation, E denotes the energy a particle carries because of its mass. The particle

Quick Revision Test COMPREHENSION Page 53


https://t.me/iitjee_toppers
Sri Chaitanya IIT Academy Physics
can also have additional energy due to its motion and its interactions with other particles. Consider a neutron
at rest, and well separated from other particles. It decays into a proton, an electron, and an undetected third
particle. 0 n1 1 H 1  1 e 0  ?

Particle Mass  C 2 ( MeV ) Kinetic energy ( MeV )

Neutron 940.97 0.00


Pr oton 939.67 0.01
Electron 0.51 0.39

The table above summarizes some data from a single neutron decay. An MeV (mega electron volt) is a unit of
energy. Column 2 shows the rest mass of the particle times the speed of light squared.

101 Assuming the table contains no major errors, what can we conclude about the (mass c2 ) of the
undetected third particle ?
A. It is 0.79MeV
B. It is 0.39MeV

s
C. It is less than or equal to 0.79MeV ; but we cannot be more precise

er
D. It is less than or equal to 0.39MeV; but we cannot be more precise

pp
Answer :D

to
Solution : D) According to the passage, subatomic reactions do not conserve mass. So, we cannot find
the third particle’s mass by setting mneutron equal to m proton  melectron  mthird particle . By constrast, the total
e_
energy in this case, the sum of mass energy and kinetic energy, is conserved. If E denotes total energy,
je

then Eneutron  E proton  Eelectron  Ethird particle


iit

The neutron has energy 949.97MeV. The proton has energy 939.67 MeV+0.01MeV=939.68MeV. The
e/

electron has energy 0.51MeV + 0.39 MeV = 0.90 MeV. Therefore, the third particle has energy
.m

Ethird particle  Eneutron  E proton  Electron = 940.97 – 939.68 – 0.90 = 0.39 MeV we just found the third
//t

particle’s total energy, the sum of its mass energy and kinetic energy. Without more information, we
s:

cannot figure out how much of that energy is mass energy.


tp

102 From the given table, which properties of the undetected third particle can we calculate?
ht

A. Total energy, but not kinetic energy

B. Kinetic energy, but not total energy

C. Both total energy and kinetic energy

D. Neither total energy nor kinetic energy

Answer :A
Solution : As just shown, energy conservation allows us to calculate the third particle’s total energy.
But we don not know what percentage of that total is mass energy vs, kinetic energy.

0
Paragraph: In figure, light of wavelength   5000 A is incident on the slits (in a horizontally fixed place).
Here, d = 1 mm and D = 1m. Take origin at O and XY plane as shown in the figure. The screen is released

Quick Revision Test COMPREHENSION Page 54


https://t.me/iitjee_toppers
Sri Chaitanya IIT Academy Physics
from rest from the initial position as shown. (Take g = 10ms-2)

0
  5000A
d
X
S1 O S2
D Y

Screen

103 The velocity of central maxima at t = 5s is

A. 50 ms-1 along Y-axis B. 50 ms-1 along X-axis

C. 25 ms-1 along Y-axis D. 3 × 10 8 ms-1 along Y-axis

Answer :A
Solution : At any time t, the situation is as shown in the figure below.

s
er
X
S1 S2

pp
gt 2
D  D1 Y 
2

to Screen
e_
P0 P
x
v = gt, a = g
je
iit

Central maxima is always lying on Y-axis at P0 . Its velocity at any time t is given by v = gt along
e/

positive Y-axis. So, required velocity is 50 m s 1 .


.m

Velocity of 2 nd maxima w.r.t central maxima at t = 2s is


//t

104
s:

A. (8 cm s 1 )i  20 m s 1 ˆj B. 8 cm s 1 iˆ
tp
ht

C. 2 cm s 1 iˆ D. 86 cm s 1 iˆ

Answer :C
Solution : Path difference corresponding to point P.

x  d sin   d tan 

dx
x 
D1

For 2nd maxima, x  2

2 D1
dx  2 D1  x 
d

Quick Revision Test COMPREHENSION Page 55


https://t.me/iitjee_toppers
Sri Chaitanya IIT Academy Physics
Location of central maxima is

 gt 2  1
 0, D   or (0, D )
 2 

Location of 2nd maxima is

 2 D1 1 
 ,D 
 d 

Velocity of 2 nd maxima w.r.t central maxima is

v2 nd 2 2
 [0  gt ]i   gt iˆ  2 cm s 1 iˆ
D d d

105 Acceleration of a 3 rd maxima w.r.t. 3rd maxima on other side of central maxima at t = 3s is

s
er
A. 0.02 m s 2 iˆ B. 0.03 m s 2 iˆ C. 10 m s 2 ˆj D. 0.6 m s 2 iˆ

pp
Answer :B to
e_
 3 D1 1 
Solution : Location of 3rd maxima is  ,D 
je

 d 
iit
e/

 3 D1 1 
Location of 3rd maxima on the other side is   ,D 
.m

 d 
//t

a3 rd  3 3 g  ˆ 6 g ˆ
i  0.03 ms 2iˆ
s:

  g  ( ) i 
3rd  d d  d
tp
ht

Paragraph:
/\/\/\/\/\/\/\/\/\/\/\/\/\/\ /\/\/\/\/\/\/\/\/\/\/\/\/\/\
\/\/\/\
\/\/\/\/\/

D
C
/\/\/\/\/\/

A
B

i i

A plane divides space into two halves. One half is filled with a homogeneous conducting medium and a
physicist works in this other. He marks the out line of a square of side ' a ' on the plane and let a current ' i ' in
and out at two of its neighbouring corners A and B using fine electrodes. The potential difference between the

Quick Revision Test COMPREHENSION Page 56


https://t.me/iitjee_toppers
Sri Chaitanya IIT Academy Physics
other two corners is found to be V as shown in figure. Using super position principle one can find resistivity
 of the conducting medium

106 The potential difference between points D and C is

i  2 1  i i  2 1
A.   B. C.   D. Zero
2 a  2  a a  2 

Answer :C
Solution : To solve this problem we can apply principle of super position. Let us assume current ‘I’
enters at A and speeds out in the form of hemispherical region. So current density at a distance r from
i
A
2 r 2
i
Now using ohm’s law, E  J  
2 r 2

s
er
Ve 2a 2a
i dr i   1  i  1 1  i  1  2 
  dV    2
 VC  VD         

pp
VD 2 a r 2  r  a 2  2 a a  2 a  2 

i  2 1 
 VC  VD   
2 a  2  to
e_
2  i  2 1  i  2 1 
je

 Net potential difference between D and C     


2 a  2   a  2 
iit
e/
.m

107 If r be the effective resistance of the battery and the connecting wires then the emf of the battery which
is supplying current into the wires will be
//t
s:

i  2  1 i
B.  ir
tp

A.    ir a
2 a  2 
ht

i   2  1 i  2  1
C.    ir D.    ir
a  2  2 a  2 

Answer :C
i  2 1 
Solution :  mf , E  V  ir     ir
 a  2 

108 The resistivity of the material is

(2  2) aV (2  2) ( 2  1) (2 2  4)
A. B.  aV C.  aV D.  aV
i i i i

Answer :A

Quick Revision Test COMPREHENSION Page 57


https://t.me/iitjee_toppers
Sri Chaitanya IIT Academy Physics

i 2 aV (2  2 ) 2 aV (2  2 ) (2  2 ) aV
Solution : V  (2  2)     
2 a i(2  2) (2  2 ) 2l i

Paragraph: Work function of metal A is equal to the ionization energy of hydrogen atom in first excited
state. Work function of metal B is equal to the ionization energy of He + ion in second orbit. Photons of same
energy E are incident on both A and B. Maximum kinetic energy of photo electrons emitted from A is twice
that of photoelectrons emitted from B.

109 Value of E (in eV) is

A. 20.8 B. 32.2 C. 24.6 D. 23.8

Answer :D
Solution : WA  ionization energy of electron in 2 nd orbit of hydrogen atom  3.4eV

s
er
WB  ionization energy of electron in the 2nd orbit of He  ion  13.6eV

pp
Now, given that K A  2 K B  E  WA  2( E  WB )  E  2WB  WA  23.8eV

to
e_
110 Radius of an electron moving in a circle in constant magnetic field is two times that of an  - particle
in the same field. Then de-Broglie wavelength of electron is x-times that of the  -particle. Here x is
je
iit

1
A. 2 B. C. 1 D. 4
e/

2
.m

Answer :C
P P P
Solution : r  r  . Given re  2 r .
//t

 Pe  2  Pe  P
Bq q 2
s:
tp

h  P
Now de-Broglie wavelength ,    e    1, x 1
ht

P  Pe

111 An electron and a proton are separated by a large distance and the electron approaches the proton with
a kinetic energy of 2eV. If the electron is captured by the proton to form a hydrogen atom in the
ground state, the wave length of photon emitted in this process will be

A. 1262.2 Å B. 793.Å C. 1204.6 Å D. 942.6 Å

Answer :B
Solution : Ei  2eV , E f  13.6eV

E  2  ( 13.6)  15.6eV

12375
( or )   793.3 Ao
15.6

Quick Revision Test COMPREHENSION Page 58


https://t.me/iitjee_toppers
Sri Chaitanya IIT Academy Physics
Paragraph: A body cools in a surrounding of constant temperature 300 C . Its heat capacity is 2 J / 0 C . Initial
temperature of the body is 40 0 C . Assume Newton’s law of cooling is valid. The body cools to 380 C in 10
minutes.

112 In further 10 min it will cool from 380 C to


A. 350 C B. 36.40 C C. 330 C D. 37.50 C
Answer :B
Solution : We have    s  ( 0   s )e  kt .

where 0  initial temperature of body = 40 0 C

 = temperature of body after time t .

Since body cools from 40 to 38 in 10 min, we have

s
38  30  (40  30)e k10 (i)

er
pp
  300  (38  30)e k10 (ii)

(i )
gives
8 10
 ,   30  6.4 to
e_
(ii )   30 8
je

  36.4 0 C
iit
e/

113 The temperature of the body in 0 C denoted by  . The variation of  versus time t is best denoted as
.m

400 C 400 C 400 C 40 0 C


//t
s:

   
30 0 C 30 0 C 300 C
tp

t t t t
ht

A. B. C. D.

Answer :A
Solution : Self-explanatory

114 When the body temperature has reached 380 C , it is heated again so that it reaches 40 0 C in 10 min. The
total heat required by the body from a heater is

A. 3.6 J B. 0.364 J C. 8 J D. 4 J

Answer :C
Solution : During heating process from 380 C to 40 0 C in 10 min, the body will lose heat in the
surrounding which will be exactly equal to the heat lost when it cooled from 400 C to 380 C in 10 min,
which is equal to ms  2  2  4 J . During heating process heat required by the body ms  4 J .
Therefore, total heat required = 8J

Quick Revision Test COMPREHENSION Page 59


https://t.me/iitjee_toppers
Sri Chaitanya IIT Academy Physics
Paragraph: A process in which work performed by an ideal gas is proportional to the corresponding
increment of its internal energy is described as a polytropic process. If we represent work done by a polytropic
process by W and increase in internal energy as U , then W  U , or

W  K1U

For this process, it can be demonstrated that the relation between pressure and volume is given by the equation

PV   K 2 (constant)

We know that a gas can have various values for molar specific heats. The molar specific heat ' C ' for an ideal
gas in polytropic process can be calculated with the help of first law of thermodynamics. In polytropic
process, the variation of molar specific heat ' C ' with  for a monoatomic gas is plotted as in the graph
shown.

s
A

er
E
F

pp
O D B 

to
e_
115 In the graph shown, the y-coordinate of point A is (for monoatomic gas)
je
iit

A. 3R / 2 B. 5 R / 2 C. 7 R / 2 D. 4R
e/

Answer :B
.m

Solution : For point A,  0


//t
s:

R(0   ) R 5
C    R
tp

(0  1)(  1)   1 2
ht

116 In the graph shown, the x-coordinate of point B is (for monoatomic gas)

A. 7/5 B. 5/3 C. 2/3 D. 8/3

Answer :B
Solution : At point B, C  0

R(   )
As C 
(  1)(  1)

   at point B

For monoatomic gas,   5 / 3

117 For a monoatomic gas, the values of polytropic constant  for which molar specific heat is negative is

Quick Revision Test COMPREHENSION Page 60


https://t.me/iitjee_toppers
Sri Chaitanya IIT Academy Physics
2 5 8 5 2 8
A.   B. 1    C. 1    D.  
3 3 3 3 3 3

Answer :C
R R
Solution : C  
 1  1

R
As  >1, is +ve always
 1

R
If   1, is –ve and ‘C’ will become +ve
 1

 For ‘C’ to become –ve,  must be greater than 1.

R R
Again  for C to become -ve

s
 1   1

er
pp
  1   1

  
to
e_
 ' C ' is –ve when 1    
je
iit
e/
.m
//t
s:
tp
ht

Quick Revision Test COMPREHENSION Page 61


https://t.me/iitjee_toppers

You might also like